0% found this document useful (0 votes)
33 views8 pages

EMC 2019 Juniors ENG Solutions

Uploaded by

Gas
Copyright
© © All Rights Reserved
We take content rights seriously. If you suspect this is your content, claim it here.
Available Formats
Download as PDF, TXT or read online on Scribd
0% found this document useful (0 votes)
33 views8 pages

EMC 2019 Juniors ENG Solutions

Uploaded by

Gas
Copyright
© © All Rights Reserved
We take content rights seriously. If you suspect this is your content, claim it here.
Available Formats
Download as PDF, TXT or read online on Scribd
You are on page 1/ 8

8th European Mathematical Cup

14th December 2019 - 22th December 2019 MLADI NADARENI MATEMATIČARI

Marin Getaldic
Junior Category

Problems and Solutions


Problem 1. Every positive integer is marked with a number from the set {0, 1, 2}, according to the following
rule:
if a positive integer k is marked with j, then the integer k + j is marked with 0.
Let S denote the sum of marks of the first 2019 positive integers. Determine the maximum possible value of S.
(Ivan Novak)

First Solution. Consider an arbitrary marking scheme which follows the given rule.

Let a denote the number of positive integers from the set {1, . . . , 2019} which are marked with a 2, b the number
of those marked with a 1, and c the number of those marked with a 0.
1 point.
We have S = 2a + b.
1 point.
For every positive integer j ∈ {1, . . . , 2017} which is marked with a 2, the number j + 2 is marked with a 0. This implies
that the number of positive integers less than 2017 marked with 2 is less than or equal to c.
1 point.
Hence, this implies a 6 c + 2. We then have

S = 2a + b 6 a + b + c + 2 = 2019 + 2 = 2021.

3 points.
Consider the following marking scheme:

210|210|210| 2200|2200|2200 . . . 2200 |22|0000 . . . .


| {z }
502 blocks of 2200

Here the i-th digit in the sequence denotes the mark of positive integer i. For this marking, S = 2021, and therefore
2021 is the maximum possible value of S.
4 points.

1
Second Solution. The marking scheme for which S = 2021 is the same as in the first solution.
4 points.
Let Sn denote the sum of marks of first n positive integers, and let ak denote the mark of k. Without loss of generality
we may assume aj = 0 for all integers j 6 0. We’ll prove the following claim by strong mathematical induction:

for every positive integer n, Sn 6 n + 2 and if equality holds, then an = 2.

1 point.
The base cases for n ∈ {1, 2} trivially hold. Suppose the claim is true for all k 6 n for some n > 2.
Suppose there exists a marking scheme for which Sn+1 > n + 4. Then if an+1 < 2, we have Sn > n + 3, which is a
contradiction. Hence, an+1 = 2.
1 point.
This implies that an ∈ {0, 2}. If an = 0, then Sn−1 > n + 2, which is a contradiction. So, an = 2.
1 point.
Now an−1 = 0 because both an and an+1 are nonzero. We now have Sn−2 > n, and by the induction hypothesis, it must
hold that Sn−2 = n and an−2 = 2. However, this is in contradiction with an being nonzero. Hence, Sn+1 6 n + 3.
1 point.
Suppose Sn+1 = n + 3 and an+1 6= 2. If an+1 = 0, then Sn > n + 3, which is a contradiction. Thus, an+1 = 1.
1 point.
Then Sn = n + 2, which implies an = 2. Then we must have an−1 = 0, and then Sn−2 = n, which implies an−2 = 2,
but an is nonzero, which is a contradiction. Therefore, the claim is true for n + 1, which implies it is true for all positive
integers. In particular, S2019 6 2021, which combined with the construction implies that the maximum value of S is
2021.
1 point.
Notes on marking:
• If a student forgets to write additional zeros beyond the first 2019 digits in his construction, but the construction
is otherwise valid, he should be awarded all 4 points for this part.
• There are many different optimal marking schemes. For example, 2200|210|210| . . . |210|22|000 . . ., where the block
|210| repeats 671 times.
• In the Second Solution, if the student writes only the first part of the induction hypothesis without the assumption
that an = 2 in the case of equality: he should be awarded 0 points, unless he reaches additional conclusions which
lead to the solution.
• In the Second Solution, if the student doesn’t comment on the base case/cases at all, he should be deducted 1
point.
• If the student proves any nontrivial lemma useful for any of the solutions, but the lemma itself isn’t worth any
points and the student wouldn’t otherwise get any of the 6 points given for proving the bound, he should get 1
point for this part.

2

Problem 2. Let (xn )n∈N be a sequence defined recursively such that x1 = 2 and
1
xn+1 = xn + for n ∈ N.
xn
Prove that the following inequality holds:

x21 x22 x22018 x22019 20192


+ + ... + + > 2 .
2x1 x2 − 1 2x2 x3 − 1 2x2018 x2019 − 1 2x2019 x2020 − 1 x2019 + x21
2019

(Ivan Novak)

First Solution. Notice that by squaring the assertion xn+1 = xn + 1


xn
we obtain the equality x2n+1 = x2n + 1
x2
+ 2 =⇒
n
2
2019
x2n + 1
x2
= x2n+1 − 2, which implies that the right hand side equals .
n x22020 − 2
1 point.
On the other hand, we have
1
2xn xn+1 − 1 = 2xn (xn + ) − 1 = 2x2n + 1.
xn
1 point.
This implies that the sum on the left hand side can be written as
1 1 1
+ + ... +
2 + x12 2 + x12 2 + x21
1 2 2019

1 point.
By squaring the given assertion, we get the equality 2 + 1
x2
= x2n+1 − x2n . This implies that the left hand side equals
n

1 1 1 1
+ 2 + ... + 2 + 2 .
x22 − x21 x3 − x22 x2019 − x22018 x2020 − x22019
1 point.
Using the inequality between arithmetic and harmonic mean, we find that the left hand side is greater than or equal to

20192
.
(x22 − x21 ) + (x23 − x22 ) + . . . + (x22020 − x22019 )
4 points.
We now notice that the denominator is a telescoping sum and it equals x22020 − x21 , which implies the right hand side
equals
20192 20192
= ,
x22020 − x21 x22020 − 2
which is exactly equal to the right hand side.
1 point.
The equality cannot hold because x22 − x21 6= x23 − x22 .
1 point.

3
Second Solution. As in the first solution, we obtain that the left hand side equals
1 1 1 1
1 + 1 + ... + + .
2 + x2 2 + x2 2 + x21 2+ x2
1
1 2 2018 2019

2 points.
Using the inequality between arithmetic and harmonic mean, we get that the left hand side is greater than or equal to

20192
1 .
2 · 2019 + x2
+ x12 + . . . + x2
1
1 2 2019

4 points.
We now prove by mathematical induction that
1 1 1
2·n+ + 2 + ... + 2 = x2n
x21 x2 xn−1

holds for every n ∈ N.


1 point.
√ 2
For n = 1, we have 2 · 1 = 2 . Suppose the claim is true for some n ∈ N. Then
1 1 1 1 1
x2n+1 = 2 + x2n + = 2 + 2n + 2 + 2 + . . . + 2 + 2,
x2n x1 x2 xn−1 xn

where we used the induction hypothesis for the last equality. This proves the claim.
2 points.
In particular, for n = 2019, we have that

20192 20192
= ,
2 · 2019 + x12 + x12 + . . . + x2
1
x22019 + x21
1 2 2019 2019

which proves the inequality.

1 1
The equality cannot hold because x2
+ 2 6= x2
+ 2.
1 2

1 point.

4
Third Solution. We prove by mathematical induction that for every n > 2 the following inequality holds:

x21 x22 x2n n2


+ + ... + > 2 .
2x1 x2 − 1 2x2 x3 − 1 2xn xn+1 − 1 xn + x12
n

2 4.5 17 4 72 17
For n = 2, the left hand side equals 5
+ 10
= 20
, and the right hand side equals 9+2 = 85
< 20
, which proves the base
2 9
case.

Suppose the claim was true for some n ∈ N. Then by the induction hypothesis, we know that

x21 x22 x2n x2n+1 n2 x2n+1


+ + ... + + > 2 1 + .
2x1 x2 − 1 2x2 x3 − 1 2xn xn+1 − 1 2xn+1 xn+2 − 1 xn + x 2 2x n+1 xn+2 − 1
n

It suffices to prove that


n2 x2n+1 (n + 1)2
1 + > 2 .
2
xn + x 2 2x x
n+1 n+2 − 1 xn+1 + x21
n n+1

1 point.
We now prove that 2xn+1 xn+2 − 1 = 2x2n+1 + 1 as in the first solution.
1 point.
We then have
n2 x2n+1 n2 x2n+1 n2 1
1 + = 2 1 + = 2 +
1
.
x2n + x2 2x x
n+1 n+2 − 1 xn + x2 2x2
n+1 + 1 xn + x12 2+ 2
n n n
xn+1
1 point.
By the inequality of arithmetic and harmonic mean, this is greater than or equal to

(n + 1)2
.
x2n + x12 + 2 + x2
1
n n+1

5 points.
1
Notice that squaring the assertion xn+1 = xn + xn
, we obtain

1
x2n + + 2 = x2n+1 .
x2n
1 point.
This implies that
(n + 1)2 (n + 1)2
= ,
x2n + x12 + 2 + x2
1
x2n+1+ x21
n n+1 n+1

which is exactly equal to the right hand side. Therefore, the claim is proven by the principle of mathematical induction.
In particular, the claim is true for n = 2019, which proves the inequality.
1 point.
Notes on marking:
• Points from separate solutions can not be added. The student should be awarded the maximum of the points scored
in the 3 presented solutions, or an appropriate number of points on an alternative solution.
• The third solution gives 5 points for the use of AM-HM inequality as opposed to 4 points in the first solution
because in the third solution it is not necessary to comment the equality case. However, if a student has n = 1 as
a basis of induction and doesn’t comment the equality case, he should be deducted 1 point out of possible 5.
• The point for proving that the equality cannot be achieved is only awarded if the student has proved the non-strict
version of inequality.

5
Problem 3. Let ABC be a triangle with circumcircle ω. Let lB and lC be two lines through the points B and
C, respectively, such that lB k lC . The second intersections of lB and lC with ω are D and E, respectively.
Assume that D and E are on the same side of BC as A. Let DA intersect lC at F and let EA intersect lB
at G. If O, O1 and O2 are circumcenters of the triangles ABC, ADG and AEF , respectively, and P is the
circumcenter of the triangle OO1 O2 , prove that lB k OP k lC .
(Stefan Lozanovski)

Sketch.

O2

E
A
S
G P

O1

B C

lB lC

Solution. Let us write ∠BAC = α, ∠ABC = β, ∠ACB = γ.

Lemma. Triangles AGD and AEF are similar to the triangle ABC.
Proof. As DBCAE is a cylic pentagon we have
∠GDA = ∠BCA = γ.
1 point.
Now from lB k lC we get that
∠DBA = ∠DBC − β = 180◦ − ∠BCE − β = α + γ − ∠BCE = α − ∠ACE
1 point.
so from the cyclicity
∠BCD = ∠BAD = 180◦ − ∠DBA − ∠ADB = 180◦ − (α − ∠ACE) − (180◦ − γ) = γ − α + ∠ACE
1 point.
Hence
∠DAG = ∠DCE = ∠BCA − ∠BCD + ∠ACE = α
1 point.
Therefore AGD is similar to the triangle ABC, and similarly for AEF .

6
Now as G, A and E are collinear and F , A and D are collinear, using Lemma we get that O, O1 and O2 are collinear.
1 point.
As O1 is the circumcenter of the triangle ADG and O1 D is the bisector of the chord AD we get that
1
∠AO1 O = ∠AO1 D = ∠AGD = β
2
and similarly ∠AO1 O = γ, so the triangle OO1 O2 is similar to the triangle ABC.
2 points.
Now as P is the circumcenter of the triangle OO1 O2 from the previous similarity we get that

∠O1 OP = ∠BAO

1 point.
Hence
∠DOP = ∠DOO1 + ∠O1 OP = ∠DBA + ∠BAO = ∠DBA + ∠ABO = ∠DBO = ∠ODB
so lB k OP k lC .
2 points.
Notes on marking:
• If a student has a partial solution with analytic methods, only points for proving facts that can be expressed in
geometric ways and lead to a compete solution can be awarded.

7
Problem 4. Let u be a positive rational number and m be a positive integer. Define a sequence q1 , q2 , q3 , . . .
such that q1 = u and for n > 2:
a a + mb
if qn−1 = for some relatively prime positive integers a and b, then qn = .
b b+1
Determine all positive integers m such that the sequence q1 , q2 , q3 , . . . is eventually periodic for any positive
rational number u.

Remark: A sequence x1 , x2 , x3 , . . . is eventually periodic if there are positive integers c and t such that xn = xn+t
for all n > c.
(Petar Nizić-Nikolac)

Solution. We will prove that the sequence is eventually periodic if and only if m is odd.
Let a1 , a2 , a3 , . . . and b1 , b2 , b3 , . . . be sequences of numerators and denumerators of q1 , q2 , q3 , . . . respectively when written
in the irreducible form, i.e. for n ∈ N:
an
qn = gcd(an , bn ) = 1
bn
Say that there was reduction in the nth step if gcd(an + mbn , bn + 1) > 1.

Case 1. m is even
Set u = 11 . Assume for the sake of contradiction that q1 , q2 , q3 , . . . is eventually periodic. Then (bn )n∈N is bounded so
there is r > 1 (pick the smallest one) such that there was reduction in the rth step. Easy to see that
r

1 m+1 3m + 1 6m + 1 10m + 1 m+1
q1 = , q 2 = , q3 = , q4 = , q5 = , . . . , qr = 2
1 2 3 4 5 r
2 points.
Now as m is even we have
! ! ! !
r r+1  m 
gcd (ar + mbr , br + 1) = gcd m + 1 + mr, r + 1 = gcd m + 1, r + 1 = gcd (r + 1)r + 1, r + 1 = 1
2 2 2

so this is a contradiction, and hence it is not eventually periodic for any positive rational number u.
1 point.
Case 2. m is odd
Assume that there is r ∈ N such that there was no reduction in the steps r, r + 1, r + 2 and r + 3. Then for i ∈ {1, 2}:

(ar+i+2 , br+i+2 ) ≡ (ar+i + mbr+i + mbr+i+1 , br+i + 1 + 1) ≡ (ar+i + 2mbr+i + m, br+i + 2) ≡ (ar+i + 1, br+i ) (mod 2)

so at least one of the following pairs: (ar+1 , br+1 ), (ar+2 , br+2 ), (ar+3 , br+3 ), (ar+4 , br+4 ) has both even entries which is
impossible (as they are coprime). Hence there was at least one reduction in the steps r, r + 1, r + 2 and r + 3.
2 points.
Therefore for all n > 1:
1
max{bn+1 , bn+2 , bn+3 , bn+4 } 6 min{bn+1 , bn+2 , bn+3 , bn+4 } + 3 6 max{bn , bn+1 , bn+2 , bn+3 } + 3
2
so there exists C > 1 such that bn 6 6 for all n > C.
2 points.
Similarly for all n > C:
1
max{an+1 , an+2 , an+3 , an+4 } 6 min{an+1 , an+2 , an+3 , an+4 } + 3 · 6m 6 max{an , an+1 , an+2 , an+3 } + 18m
2
so there exists D > C such that an 6 36m for all n > D.
2 points.
We conclude that for all n > D there are finitely many pairs (6 · 36m = 216m) that (an , bn ) attains so it becomes
eventually periodic for any positive rational number u.
1 point.
Notes on marking:
• Case 1 and Case 2 are always worth 3 points and 7 points respectively.

You might also like

pFad - Phonifier reborn

Pfad - The Proxy pFad of © 2024 Garber Painting. All rights reserved.

Note: This service is not intended for secure transactions such as banking, social media, email, or purchasing. Use at your own risk. We assume no liability whatsoever for broken pages.


Alternative Proxies:

Alternative Proxy

pFad Proxy

pFad v3 Proxy

pFad v4 Proxy